Evaluation of $sumlimits_{k=1}^nleft(x^{k}+frac{1}{x^k}right)^k$












4












$begingroup$


Can anyone find a simplified expression for the sum $displaystyle sum_{k=1}^nleft(x^{k}+frac{1}{x^k}right)^k$? I have tried expanding the first few terms but it gets a little messy with no clear leads. I suspect formulae for geometric series may come into it somehow, but at the moment it isn't clear how to start.










share|cite|improve this question











$endgroup$












  • $begingroup$
    It's from an Olympiad paper I believe, so I assumed there would be a neat solution, with no need for double sums etc.
    $endgroup$
    – wrb98
    Mar 8 '17 at 20:46








  • 2




    $begingroup$
    I honestly have no clue. After binomial expansion, you end up with things like $x^{k^2}$, which have no closed form sum.
    $endgroup$
    – Simply Beautiful Art
    Mar 8 '17 at 20:47










  • $begingroup$
    Exactly. I tried to see if expanding the first brackets could lead to some pattern and then proceed by induction, but it's a dead end.
    $endgroup$
    – wrb98
    Mar 8 '17 at 20:49






  • 1




    $begingroup$
    Interesting, even Wolfram|Alpha can't figure it out.
    $endgroup$
    – projectilemotion
    Mar 8 '17 at 20:52






  • 3




    $begingroup$
    @Will: is the original problem really about simplifying such sum, or just about evaluating the coefficient of $x^0$ in the Laurent series?
    $endgroup$
    – Jack D'Aurizio
    Mar 8 '17 at 21:06
















4












$begingroup$


Can anyone find a simplified expression for the sum $displaystyle sum_{k=1}^nleft(x^{k}+frac{1}{x^k}right)^k$? I have tried expanding the first few terms but it gets a little messy with no clear leads. I suspect formulae for geometric series may come into it somehow, but at the moment it isn't clear how to start.










share|cite|improve this question











$endgroup$












  • $begingroup$
    It's from an Olympiad paper I believe, so I assumed there would be a neat solution, with no need for double sums etc.
    $endgroup$
    – wrb98
    Mar 8 '17 at 20:46








  • 2




    $begingroup$
    I honestly have no clue. After binomial expansion, you end up with things like $x^{k^2}$, which have no closed form sum.
    $endgroup$
    – Simply Beautiful Art
    Mar 8 '17 at 20:47










  • $begingroup$
    Exactly. I tried to see if expanding the first brackets could lead to some pattern and then proceed by induction, but it's a dead end.
    $endgroup$
    – wrb98
    Mar 8 '17 at 20:49






  • 1




    $begingroup$
    Interesting, even Wolfram|Alpha can't figure it out.
    $endgroup$
    – projectilemotion
    Mar 8 '17 at 20:52






  • 3




    $begingroup$
    @Will: is the original problem really about simplifying such sum, or just about evaluating the coefficient of $x^0$ in the Laurent series?
    $endgroup$
    – Jack D'Aurizio
    Mar 8 '17 at 21:06














4












4








4


6



$begingroup$


Can anyone find a simplified expression for the sum $displaystyle sum_{k=1}^nleft(x^{k}+frac{1}{x^k}right)^k$? I have tried expanding the first few terms but it gets a little messy with no clear leads. I suspect formulae for geometric series may come into it somehow, but at the moment it isn't clear how to start.










share|cite|improve this question











$endgroup$




Can anyone find a simplified expression for the sum $displaystyle sum_{k=1}^nleft(x^{k}+frac{1}{x^k}right)^k$? I have tried expanding the first few terms but it gets a little messy with no clear leads. I suspect formulae for geometric series may come into it somehow, but at the moment it isn't clear how to start.







sequences-and-series algebra-precalculus binomial-theorem






share|cite|improve this question















share|cite|improve this question













share|cite|improve this question




share|cite|improve this question








edited Jan 18 at 12:51









Martin Sleziak

44.7k10119272




44.7k10119272










asked Mar 8 '17 at 20:44









wrb98wrb98

579313




579313












  • $begingroup$
    It's from an Olympiad paper I believe, so I assumed there would be a neat solution, with no need for double sums etc.
    $endgroup$
    – wrb98
    Mar 8 '17 at 20:46








  • 2




    $begingroup$
    I honestly have no clue. After binomial expansion, you end up with things like $x^{k^2}$, which have no closed form sum.
    $endgroup$
    – Simply Beautiful Art
    Mar 8 '17 at 20:47










  • $begingroup$
    Exactly. I tried to see if expanding the first brackets could lead to some pattern and then proceed by induction, but it's a dead end.
    $endgroup$
    – wrb98
    Mar 8 '17 at 20:49






  • 1




    $begingroup$
    Interesting, even Wolfram|Alpha can't figure it out.
    $endgroup$
    – projectilemotion
    Mar 8 '17 at 20:52






  • 3




    $begingroup$
    @Will: is the original problem really about simplifying such sum, or just about evaluating the coefficient of $x^0$ in the Laurent series?
    $endgroup$
    – Jack D'Aurizio
    Mar 8 '17 at 21:06


















  • $begingroup$
    It's from an Olympiad paper I believe, so I assumed there would be a neat solution, with no need for double sums etc.
    $endgroup$
    – wrb98
    Mar 8 '17 at 20:46








  • 2




    $begingroup$
    I honestly have no clue. After binomial expansion, you end up with things like $x^{k^2}$, which have no closed form sum.
    $endgroup$
    – Simply Beautiful Art
    Mar 8 '17 at 20:47










  • $begingroup$
    Exactly. I tried to see if expanding the first brackets could lead to some pattern and then proceed by induction, but it's a dead end.
    $endgroup$
    – wrb98
    Mar 8 '17 at 20:49






  • 1




    $begingroup$
    Interesting, even Wolfram|Alpha can't figure it out.
    $endgroup$
    – projectilemotion
    Mar 8 '17 at 20:52






  • 3




    $begingroup$
    @Will: is the original problem really about simplifying such sum, or just about evaluating the coefficient of $x^0$ in the Laurent series?
    $endgroup$
    – Jack D'Aurizio
    Mar 8 '17 at 21:06
















$begingroup$
It's from an Olympiad paper I believe, so I assumed there would be a neat solution, with no need for double sums etc.
$endgroup$
– wrb98
Mar 8 '17 at 20:46






$begingroup$
It's from an Olympiad paper I believe, so I assumed there would be a neat solution, with no need for double sums etc.
$endgroup$
– wrb98
Mar 8 '17 at 20:46






2




2




$begingroup$
I honestly have no clue. After binomial expansion, you end up with things like $x^{k^2}$, which have no closed form sum.
$endgroup$
– Simply Beautiful Art
Mar 8 '17 at 20:47




$begingroup$
I honestly have no clue. After binomial expansion, you end up with things like $x^{k^2}$, which have no closed form sum.
$endgroup$
– Simply Beautiful Art
Mar 8 '17 at 20:47












$begingroup$
Exactly. I tried to see if expanding the first brackets could lead to some pattern and then proceed by induction, but it's a dead end.
$endgroup$
– wrb98
Mar 8 '17 at 20:49




$begingroup$
Exactly. I tried to see if expanding the first brackets could lead to some pattern and then proceed by induction, but it's a dead end.
$endgroup$
– wrb98
Mar 8 '17 at 20:49




1




1




$begingroup$
Interesting, even Wolfram|Alpha can't figure it out.
$endgroup$
– projectilemotion
Mar 8 '17 at 20:52




$begingroup$
Interesting, even Wolfram|Alpha can't figure it out.
$endgroup$
– projectilemotion
Mar 8 '17 at 20:52




3




3




$begingroup$
@Will: is the original problem really about simplifying such sum, or just about evaluating the coefficient of $x^0$ in the Laurent series?
$endgroup$
– Jack D'Aurizio
Mar 8 '17 at 21:06




$begingroup$
@Will: is the original problem really about simplifying such sum, or just about evaluating the coefficient of $x^0$ in the Laurent series?
$endgroup$
– Jack D'Aurizio
Mar 8 '17 at 21:06










2 Answers
2






active

oldest

votes


















3












$begingroup$

Let $S(n)$ be your sum.
For $1 le m le n^2$, the coefficient of $x^m$ (and of $x^{-m}$, by symmetry) in $S(n)$ is
$$ [x^m]; S(n) = sum_k {k choose frac{m}{2k}+frac{k}{2}}$$
where the sum is over all divisors $k$ of $m$ such that $m le k^2 le n^2$
and $frac{m}{k} equiv k mod 2$. In particular this is $0$ if $m equiv 2 mod 4$.



The coefficient of $x^0$ in $S(n)$ is $$ [x^0] ; S(n) = sum_{j=1}^{lfloor n/2 rfloor} {2j choose j}$$






share|cite|improve this answer











$endgroup$













  • $begingroup$
    My guess, based on the usual approximation to $binom{2j}{j}$, is that $sum_{j=1}^m binom{2j}{j} approx dfrac{4^m}{sqrt{pi }}dfrac{4}{3sqrt{m}}$.
    $endgroup$
    – marty cohen
    Mar 9 '17 at 3:23






  • 1




    $begingroup$
    @martycohen See OEIS sequence A066796, in particular the formula by Vaclav Kotesovec, which agrees with your guess.
    $endgroup$
    – Robert Israel
    Mar 9 '17 at 4:53





















0












$begingroup$

Not an answer, just a long comment;



Not sure how to proceed, but one way to rewrite it is $x=e^{iphi}$, getting



$$sum_{k=1}^n (2cos kphi)^k=sum_{k=1}^n 2^k T_k^k(cos(phi))$$
where $T$ is a Chebyshev polynomial of the first kind.



It seems to me that the expression is too convoluted to expect a nice solution. Another similar approach that might be considered is to express everything with $x+x^{-1}$. We have expressions of the type



$$(x+x^{-1})^2=2+(x^2+x^{-2})$$
$$(x+x^{-1})^3=3(x+x^{-1})+(x^3+x^{-3})$$
which could be solved for $x^k+x^{-k}$ but I can't see how to get a closed form for a general term. This solution does give you the constant term (reproduces result by @RobertIsrael) but the rest gets complicated due to $()^k$ - we get summations over very strange sets.



A reference that might be useful - it includes some very similar expressions.






share|cite|improve this answer









$endgroup$













    Your Answer





    StackExchange.ifUsing("editor", function () {
    return StackExchange.using("mathjaxEditing", function () {
    StackExchange.MarkdownEditor.creationCallbacks.add(function (editor, postfix) {
    StackExchange.mathjaxEditing.prepareWmdForMathJax(editor, postfix, [["$", "$"], ["\\(","\\)"]]);
    });
    });
    }, "mathjax-editing");

    StackExchange.ready(function() {
    var channelOptions = {
    tags: "".split(" "),
    id: "69"
    };
    initTagRenderer("".split(" "), "".split(" "), channelOptions);

    StackExchange.using("externalEditor", function() {
    // Have to fire editor after snippets, if snippets enabled
    if (StackExchange.settings.snippets.snippetsEnabled) {
    StackExchange.using("snippets", function() {
    createEditor();
    });
    }
    else {
    createEditor();
    }
    });

    function createEditor() {
    StackExchange.prepareEditor({
    heartbeatType: 'answer',
    autoActivateHeartbeat: false,
    convertImagesToLinks: true,
    noModals: true,
    showLowRepImageUploadWarning: true,
    reputationToPostImages: 10,
    bindNavPrevention: true,
    postfix: "",
    imageUploader: {
    brandingHtml: "Powered by u003ca class="icon-imgur-white" href="https://imgur.com/"u003eu003c/au003e",
    contentPolicyHtml: "User contributions licensed under u003ca href="https://creativecommons.org/licenses/by-sa/3.0/"u003ecc by-sa 3.0 with attribution requiredu003c/au003e u003ca href="https://stackoverflow.com/legal/content-policy"u003e(content policy)u003c/au003e",
    allowUrls: true
    },
    noCode: true, onDemand: true,
    discardSelector: ".discard-answer"
    ,immediatelyShowMarkdownHelp:true
    });


    }
    });














    draft saved

    draft discarded


















    StackExchange.ready(
    function () {
    StackExchange.openid.initPostLogin('.new-post-login', 'https%3a%2f%2fmath.stackexchange.com%2fquestions%2f2178172%2fevaluation-of-sum-limits-k-1n-leftxk-frac1xk-rightk%23new-answer', 'question_page');
    }
    );

    Post as a guest















    Required, but never shown

























    2 Answers
    2






    active

    oldest

    votes








    2 Answers
    2






    active

    oldest

    votes









    active

    oldest

    votes






    active

    oldest

    votes









    3












    $begingroup$

    Let $S(n)$ be your sum.
    For $1 le m le n^2$, the coefficient of $x^m$ (and of $x^{-m}$, by symmetry) in $S(n)$ is
    $$ [x^m]; S(n) = sum_k {k choose frac{m}{2k}+frac{k}{2}}$$
    where the sum is over all divisors $k$ of $m$ such that $m le k^2 le n^2$
    and $frac{m}{k} equiv k mod 2$. In particular this is $0$ if $m equiv 2 mod 4$.



    The coefficient of $x^0$ in $S(n)$ is $$ [x^0] ; S(n) = sum_{j=1}^{lfloor n/2 rfloor} {2j choose j}$$






    share|cite|improve this answer











    $endgroup$













    • $begingroup$
      My guess, based on the usual approximation to $binom{2j}{j}$, is that $sum_{j=1}^m binom{2j}{j} approx dfrac{4^m}{sqrt{pi }}dfrac{4}{3sqrt{m}}$.
      $endgroup$
      – marty cohen
      Mar 9 '17 at 3:23






    • 1




      $begingroup$
      @martycohen See OEIS sequence A066796, in particular the formula by Vaclav Kotesovec, which agrees with your guess.
      $endgroup$
      – Robert Israel
      Mar 9 '17 at 4:53


















    3












    $begingroup$

    Let $S(n)$ be your sum.
    For $1 le m le n^2$, the coefficient of $x^m$ (and of $x^{-m}$, by symmetry) in $S(n)$ is
    $$ [x^m]; S(n) = sum_k {k choose frac{m}{2k}+frac{k}{2}}$$
    where the sum is over all divisors $k$ of $m$ such that $m le k^2 le n^2$
    and $frac{m}{k} equiv k mod 2$. In particular this is $0$ if $m equiv 2 mod 4$.



    The coefficient of $x^0$ in $S(n)$ is $$ [x^0] ; S(n) = sum_{j=1}^{lfloor n/2 rfloor} {2j choose j}$$






    share|cite|improve this answer











    $endgroup$













    • $begingroup$
      My guess, based on the usual approximation to $binom{2j}{j}$, is that $sum_{j=1}^m binom{2j}{j} approx dfrac{4^m}{sqrt{pi }}dfrac{4}{3sqrt{m}}$.
      $endgroup$
      – marty cohen
      Mar 9 '17 at 3:23






    • 1




      $begingroup$
      @martycohen See OEIS sequence A066796, in particular the formula by Vaclav Kotesovec, which agrees with your guess.
      $endgroup$
      – Robert Israel
      Mar 9 '17 at 4:53
















    3












    3








    3





    $begingroup$

    Let $S(n)$ be your sum.
    For $1 le m le n^2$, the coefficient of $x^m$ (and of $x^{-m}$, by symmetry) in $S(n)$ is
    $$ [x^m]; S(n) = sum_k {k choose frac{m}{2k}+frac{k}{2}}$$
    where the sum is over all divisors $k$ of $m$ such that $m le k^2 le n^2$
    and $frac{m}{k} equiv k mod 2$. In particular this is $0$ if $m equiv 2 mod 4$.



    The coefficient of $x^0$ in $S(n)$ is $$ [x^0] ; S(n) = sum_{j=1}^{lfloor n/2 rfloor} {2j choose j}$$






    share|cite|improve this answer











    $endgroup$



    Let $S(n)$ be your sum.
    For $1 le m le n^2$, the coefficient of $x^m$ (and of $x^{-m}$, by symmetry) in $S(n)$ is
    $$ [x^m]; S(n) = sum_k {k choose frac{m}{2k}+frac{k}{2}}$$
    where the sum is over all divisors $k$ of $m$ such that $m le k^2 le n^2$
    and $frac{m}{k} equiv k mod 2$. In particular this is $0$ if $m equiv 2 mod 4$.



    The coefficient of $x^0$ in $S(n)$ is $$ [x^0] ; S(n) = sum_{j=1}^{lfloor n/2 rfloor} {2j choose j}$$







    share|cite|improve this answer














    share|cite|improve this answer



    share|cite|improve this answer








    edited Mar 8 '17 at 21:37

























    answered Mar 8 '17 at 21:30









    Robert IsraelRobert Israel

    325k23214468




    325k23214468












    • $begingroup$
      My guess, based on the usual approximation to $binom{2j}{j}$, is that $sum_{j=1}^m binom{2j}{j} approx dfrac{4^m}{sqrt{pi }}dfrac{4}{3sqrt{m}}$.
      $endgroup$
      – marty cohen
      Mar 9 '17 at 3:23






    • 1




      $begingroup$
      @martycohen See OEIS sequence A066796, in particular the formula by Vaclav Kotesovec, which agrees with your guess.
      $endgroup$
      – Robert Israel
      Mar 9 '17 at 4:53




















    • $begingroup$
      My guess, based on the usual approximation to $binom{2j}{j}$, is that $sum_{j=1}^m binom{2j}{j} approx dfrac{4^m}{sqrt{pi }}dfrac{4}{3sqrt{m}}$.
      $endgroup$
      – marty cohen
      Mar 9 '17 at 3:23






    • 1




      $begingroup$
      @martycohen See OEIS sequence A066796, in particular the formula by Vaclav Kotesovec, which agrees with your guess.
      $endgroup$
      – Robert Israel
      Mar 9 '17 at 4:53


















    $begingroup$
    My guess, based on the usual approximation to $binom{2j}{j}$, is that $sum_{j=1}^m binom{2j}{j} approx dfrac{4^m}{sqrt{pi }}dfrac{4}{3sqrt{m}}$.
    $endgroup$
    – marty cohen
    Mar 9 '17 at 3:23




    $begingroup$
    My guess, based on the usual approximation to $binom{2j}{j}$, is that $sum_{j=1}^m binom{2j}{j} approx dfrac{4^m}{sqrt{pi }}dfrac{4}{3sqrt{m}}$.
    $endgroup$
    – marty cohen
    Mar 9 '17 at 3:23




    1




    1




    $begingroup$
    @martycohen See OEIS sequence A066796, in particular the formula by Vaclav Kotesovec, which agrees with your guess.
    $endgroup$
    – Robert Israel
    Mar 9 '17 at 4:53






    $begingroup$
    @martycohen See OEIS sequence A066796, in particular the formula by Vaclav Kotesovec, which agrees with your guess.
    $endgroup$
    – Robert Israel
    Mar 9 '17 at 4:53













    0












    $begingroup$

    Not an answer, just a long comment;



    Not sure how to proceed, but one way to rewrite it is $x=e^{iphi}$, getting



    $$sum_{k=1}^n (2cos kphi)^k=sum_{k=1}^n 2^k T_k^k(cos(phi))$$
    where $T$ is a Chebyshev polynomial of the first kind.



    It seems to me that the expression is too convoluted to expect a nice solution. Another similar approach that might be considered is to express everything with $x+x^{-1}$. We have expressions of the type



    $$(x+x^{-1})^2=2+(x^2+x^{-2})$$
    $$(x+x^{-1})^3=3(x+x^{-1})+(x^3+x^{-3})$$
    which could be solved for $x^k+x^{-k}$ but I can't see how to get a closed form for a general term. This solution does give you the constant term (reproduces result by @RobertIsrael) but the rest gets complicated due to $()^k$ - we get summations over very strange sets.



    A reference that might be useful - it includes some very similar expressions.






    share|cite|improve this answer









    $endgroup$


















      0












      $begingroup$

      Not an answer, just a long comment;



      Not sure how to proceed, but one way to rewrite it is $x=e^{iphi}$, getting



      $$sum_{k=1}^n (2cos kphi)^k=sum_{k=1}^n 2^k T_k^k(cos(phi))$$
      where $T$ is a Chebyshev polynomial of the first kind.



      It seems to me that the expression is too convoluted to expect a nice solution. Another similar approach that might be considered is to express everything with $x+x^{-1}$. We have expressions of the type



      $$(x+x^{-1})^2=2+(x^2+x^{-2})$$
      $$(x+x^{-1})^3=3(x+x^{-1})+(x^3+x^{-3})$$
      which could be solved for $x^k+x^{-k}$ but I can't see how to get a closed form for a general term. This solution does give you the constant term (reproduces result by @RobertIsrael) but the rest gets complicated due to $()^k$ - we get summations over very strange sets.



      A reference that might be useful - it includes some very similar expressions.






      share|cite|improve this answer









      $endgroup$
















        0












        0








        0





        $begingroup$

        Not an answer, just a long comment;



        Not sure how to proceed, but one way to rewrite it is $x=e^{iphi}$, getting



        $$sum_{k=1}^n (2cos kphi)^k=sum_{k=1}^n 2^k T_k^k(cos(phi))$$
        where $T$ is a Chebyshev polynomial of the first kind.



        It seems to me that the expression is too convoluted to expect a nice solution. Another similar approach that might be considered is to express everything with $x+x^{-1}$. We have expressions of the type



        $$(x+x^{-1})^2=2+(x^2+x^{-2})$$
        $$(x+x^{-1})^3=3(x+x^{-1})+(x^3+x^{-3})$$
        which could be solved for $x^k+x^{-k}$ but I can't see how to get a closed form for a general term. This solution does give you the constant term (reproduces result by @RobertIsrael) but the rest gets complicated due to $()^k$ - we get summations over very strange sets.



        A reference that might be useful - it includes some very similar expressions.






        share|cite|improve this answer









        $endgroup$



        Not an answer, just a long comment;



        Not sure how to proceed, but one way to rewrite it is $x=e^{iphi}$, getting



        $$sum_{k=1}^n (2cos kphi)^k=sum_{k=1}^n 2^k T_k^k(cos(phi))$$
        where $T$ is a Chebyshev polynomial of the first kind.



        It seems to me that the expression is too convoluted to expect a nice solution. Another similar approach that might be considered is to express everything with $x+x^{-1}$. We have expressions of the type



        $$(x+x^{-1})^2=2+(x^2+x^{-2})$$
        $$(x+x^{-1})^3=3(x+x^{-1})+(x^3+x^{-3})$$
        which could be solved for $x^k+x^{-k}$ but I can't see how to get a closed form for a general term. This solution does give you the constant term (reproduces result by @RobertIsrael) but the rest gets complicated due to $()^k$ - we get summations over very strange sets.



        A reference that might be useful - it includes some very similar expressions.







        share|cite|improve this answer












        share|cite|improve this answer



        share|cite|improve this answer










        answered Jan 18 at 14:26









        orionorion

        13.6k11837




        13.6k11837






























            draft saved

            draft discarded




















































            Thanks for contributing an answer to Mathematics Stack Exchange!


            • Please be sure to answer the question. Provide details and share your research!

            But avoid



            • Asking for help, clarification, or responding to other answers.

            • Making statements based on opinion; back them up with references or personal experience.


            Use MathJax to format equations. MathJax reference.


            To learn more, see our tips on writing great answers.




            draft saved


            draft discarded














            StackExchange.ready(
            function () {
            StackExchange.openid.initPostLogin('.new-post-login', 'https%3a%2f%2fmath.stackexchange.com%2fquestions%2f2178172%2fevaluation-of-sum-limits-k-1n-leftxk-frac1xk-rightk%23new-answer', 'question_page');
            }
            );

            Post as a guest















            Required, but never shown





















































            Required, but never shown














            Required, but never shown












            Required, but never shown







            Required, but never shown

































            Required, but never shown














            Required, but never shown












            Required, but never shown







            Required, but never shown







            Popular posts from this blog

            Can a sorcerer learn a 5th-level spell early by creating spell slots using the Font of Magic feature?

            Does disintegrating a polymorphed enemy still kill it after the 2018 errata?

            A Topological Invariant for $pi_3(U(n))$